BAC BLANC DE MATHEMATIQUES EN TM1 et TM2 .

Transcription

BAC BLANC DE MATHEMATIQUES EN TM1 et TM2 .
BAC BLANC DE MATHEMATIQUES
EN TM1 et TM2 .
L’ordre des exercices n’a pas d’importance.
La clarté de la rédaction et des raisonnements interviendront pour une part importante dans l’appréciation
des copies. La calculatrice est autorisée.
I ( 5 points)
Un propriétaire propose à partir du 1er janvier 2000 un appartement dont le loyer annuel initial est 6 000 €.
Il envisage deux types d'augmentation :
1°) Dans le premier cas, le loyer annuel augmenterait chaque année de 200 €.
On désigne par Pn le montant annuel du loyer pour l'année (2000 + n). On a donc P0 = 6 000.
a. Calculer Pl et P2.
b. Montrer que (Pn ) est une suite arithmétique. Déterminer sa raison.
c. Exprimer Pn en fonction de n.
d. Quel serait le montant annuel du loyer en 2015, arrondi à l'euro près ?
e. En quelle année le loyer dépassera-t-il le double du loyer initial ?
f. Calculer le montant total des loyers versés de 2000 à 2015 compris.
2°) Dans le deuxième cas, le loyer annuel augmenterait de chaque année de 3 %.
On désigne par Qn le montant annuel du loyer pour l'année (2000 + n). On a donc Q0 = 6 000.
a. Calculer Q1 et Q2 .
b. Montrer que ( Qn ) est une suite géométrique. Déterminer sa raison.
c. Exprimer Qn en fonction de n.
d. Quel serait le montant annuel du loyer en 2015, arrondi à l'euro près ?
e. En quelle année le loyer dépassera-t-il le double du loyer initial ?
f. Calculer le montant total des loyers versés de 2000 à 2015 compris.
II ( 5 points)
Pour chacune des trois questions de ce questionnaire à choix multiple (QCM), une seule des trois propositions
est exacte .Le candidat indiquera sur sa copie le numéro de la question et la lettre correspondant à la réponse
choisie. Pour chaque question, il est compté un point si la réponse est exacte, -0,5 si la réponse est fausse, zéro
sinon. Aucune justification n'est demandée.
1°) Un prix T.T.C. est de 129,90 € avec une T.V.A. à 19,6 %. Le prix H.T.arrondi au centime est de :
a. 155,36 €;
b. 104,40 €;
c. 108,61 €.
2°) Le prix d'un produit augmente de 8 %, puis diminue de 7 %. Finalement la variation est :
a. une augmentation de 0,44 % ; b. une diminution de 1 % ; c. une augmentation de 1 %.
3°) Si 3 400 a pour indice 100, quel est l'indice de4318 ?
a. 79 ;
b. 127 ;
c. 27 %.
4°) Le volume d'un ballon publicitaire a augmenté de 60 % sous l’effet de la chaleur.
Pour retrouver son volume initial il doit maintenant diminuer de :
a. 40 % ;
b. 37,5 % ;
c. 60 %.
5°) Entre le 01/01/2000 et le 01/01/2005 le coût de la vie a augmenté de 17%. Cela correspond à une
hausse annuelle moyenne arrondie au centième, de :
a. 3,4 % ;
b. 3 % ;
c.3,19 %.
-1–
Terminale Mercatique
http://mathemitec.free.fr/index.php
Bac Blanc 01
2008-2009
III ( 5 points)
En 1990, une entreprise de fabrication de jouets a été créée. Le but de cet exercice est d'étudier l'évolution du
pourcentage des salariés travaillant à temps partiel par rapport au total des salariés de l'entreprise.
Le tableau suivant donne, pour les années indiquées, le nombre x d'années écoulées depuis 1990 et le
pourcentage y de salariés à temps partiel correspondant.
Années
x
y (en %)
1992
2
8,9
1994
4
10,2
1995
5
10,5
1998
8
12,2
1999
9
12,3
2001
11
13,2
2002
12
13,8
2003
13
14,9
1°) Dans un repère orthonormal (O ; i, j ) d'unité graphique 1 cm, représenter le nuage des points M de
coordonnées (x ; y).
2°) Déterminer les coordonnées du point moyen G de ce nuage et le placer sur le graphique précédent.
3°) Déterminer les coordonnées du point moyen G1 du nuage formé des quatre premiers points et
placer ce point sur le graphique.
4°) Déterminer les coordonnées du point moyen G2 du nuage formé des quatre dernier points et placer
ce point sur le graphique.
5°) Déterminer l’équation de la droite (G1G2). La droite (G1G2) s’appelle la droite de Mayer.
6°) Vérifier que le point G appartient à (G1G2).
7°) Tracer cette droite sur le graphique précédent.
8°) En utilisant l’ajustement affine précédent, quel sera le pourcentage de salariés travaillant à
temps partiel par rapport au total des salariés de l'entreprise en 2007 ?
9°) En utilisant l’ajustement affine précédent, déterminer en quelle année le pourcentage de salariés
travaillant à temps partiel par rapport au total des salariés sera d’au moins 20 % ?
10°) Avec votre calculette trouver l’équation de la droite de régression par la méthode des moindres
carrés de y en x. On donnera les résultats à 0,01près. Comparer avec 5°).
IV ( 5 points)
Partie A : Étude de deux fonctions
1°) Soit f la fonction définie sur [0,4] par : f ( x) = 3 + ln(2 x + 2) .
1
. En déduire le signe de f '( x) .Etablir le tableau de variation de f .
x +1
2°) Soit g la fonction définie sur [1,4] par : g ( x ) = x ² − 4 x + 6 .
a. Calculer g '( x ) . En déduire le signe de g '( x ) . Etablir le tableau de variation de g .
a. Montrer que f '( x) =
b. Recopier et compléter le tableau de valeur suivant ; arrondir à 10−2 .
x
f ( x)
g ( x)
0
0,5
1
1,5
2
2,5
3
3,5
4
A l’aide du tableau construire la courbe représentative Cf de f et la courbe représentative Cg de g dans un
repère orthonormal : unité graphique est 2 cm pour une unité sur les deux axes.
Partie B : Application économique
Une entreprise fabrique un certain type de pièces pour les téléphones mobiles. On admet que pour x milliers de
pièces fabriquées et vendues : la recette, en milliers d'euros, est f ( x ) et le coût total de production, en milliers
d'euros, est g ( x ) .
1°) Déterminer graphiquement sur quel intervalle l'entreprise réalise un bénéfice.
2°) Déterminer graphiquement une valeur approchée de la production x0 pour laquelle le bénéfice est maximal.
3°) A l’aide du tableau donner une valeur approchée du bénéfice maximal. Arrondir à la dizaine d’euro.
-2–
Terminale Mercatique
http://mathemitec.free.fr/index.php
Bac Blanc 01
2008-2009
Corrigé
BAC BLANC DE MATHEMATIQUES, Terminale Mercatique
I
1a. D’après l’énoncé P1 = P0 + 200 = 6200 et P2 = P1 + 200 = 6400 .
1b. Pour passer d’un terme au suivant on ajoute toujours 200€, donc la suite est arithmétique de raison 200 et de
premier terme 6000.
1c. On sait alors que Pn = P0 + nr cad Pn = 6000 + 200n .
1d. Comme Pn correspond à 2000+n, c’est P15 qui correspond à 2015. On a P15 = 6000 + 15 × 200 = 9000€ .
1e. On cherche à déterminer P0 + P1 + ... + P15 = nbre de termes ×
1er + dernier
6000 + 9000
= 16 ×
= 120 000€ .
2
2
2a. Pour augmenter un nombre de 3% on le multiplie par 1 +
3
= 1.03.
100
Ainsi, Q0 = 1.03 × 6000 = 6180 , Q1 = 1.03 × 6180 = 6365.4
2b. Pour passer d’un terme au suivant on multiplie toujours 1.03, donc la suite est géométrique de raison 1.03 et
de premier terme 6000.
2c. On a alors Qn = Q0 × q n cad Qn = 6000 × 1.03n .
2d. En 2015, le loyer sera de Q15 = 6000 × 1.0315 ≈ 9348€ (arrondi à l’euro).
2e. On cherche à déterminer Q0 + Q1 + ... + Q15 = 1er terme ×
1 − q nbre de terme
1 − 1.0316
= 6000
≈ 120941 €.
1− q
1 − 1.03
II
1°) Un prix T.T.C. est de 129,90 € avec une T.V.A. à 19,6 %. Le prix H.T.arrondi au centime est de :
a. 155,36 €;
b. 104,40 €;
c. 108,61 €.
Comme PTTC = 1.196 PHT on a PHT =
PTTC
≈ 108.61€ .
1.196
2°) Le prix d'un produit augmente de 8 %, puis diminue de 7 %. Finalement la variation est :
a. une augmentation de 0,44 % ; b. une diminution de 1 % ; c. une augmentation de 1 %.
Le coefficient multiplicateur global est de 1.08 × 0.93 = 1.0044 qui correspond a une hausse de 0.44%.
3°) Si 3 400 a pour indice 100, quel est l'indice de4318 ?
a. 79 ;
b. 127 ;
On a
3400 4318
100
I
donc I =
c. 27 %.
4318 × 100
= 127 .
3400
4°) Le volume d'un ballon publicitaire a augmenté de 60 % sous l’effet de la chaleur.
Pour retrouver son volume initial il doit maintenant diminuer de :
a. 40 % ;
b. 37,5 % ;
c. 60 %.
On a V f = 1.6Vi ⇒ Vi =
1
1
Vi or
= 0.625 donc Vi = 0.625Vi et multiplier par 0.625 revient à appliquer une
1.6
1.6
baisse de 1-0.625 = 37.5 %
5°) Entre le 01/01/2000 et le 01/01/2005 le coût de la vie a augmenté de 17%. Cela correspond à une
hausse annuelle moyenne arrondie au centième, de :
a. 3,4 % ;
b. 3 % ;
c.3,19 %.
1
On a (1 + tm )5 = 1.17 ⇒ tm = 1.17 5 − 1 ≈ 3.19% .
Page 3 sur 6
III
Années
x
y (en %)
1992
2
8,9
1994
4
10,2
1995
5
10,5
1998
8
12,2
1999
9
12,3
2001
11
13,2
2002
12
13,8
2003
13
14,9
1°) Voir figure jointe.
2°) Le point moyen a pour coordonnées la moyenne des abscisse et la moyenne des ordonnées. On a donc
G ( 8;12 ) .
3°) De même, on a G1 ( 4.75;10.45 ) .
4°) Et on a G2 (11.25;13.55 ) .
5°) La droite est non verticale donc son équation est du type y = ax + b, avec :
> a=
yG2 − yG1
xG2 − xG1
=
13.55 − 10.45
≈ 0.48 .
11.25 − 4.75
> d’où y = 0.48x + b et comme G1 ( 4.75;10.45 ) est sur la droite, ses coordonnées vérifient l’équation
d’où 10.45 = 0.48 × 4.75 + b ⇒ b = 10.45 − 0.48 × 4.75 = 8.17
> ainsi, l’équation réduite de la droite (G1G2) est y = 0.48x + 8.17, où les coefficients ont été arrondis
au centième.
6°) Remplaçons l’abscisse de G dans l’équation : 0.48 × 8 + 8.17 = 12.01 ≠ yG donc il semble que non.
Cependant, les erreurs d’arrondis nous ont peut être trompés… G est le point moyen du nuage donc
par construction ,c’est le milieu de [ G1G2 ] donc il est bien sur la droite ( G1G2 ) !
7°) Voir figure.
8°) 2007 correspond à x = 17 donc y = 0.48 × 17 + 8.17 ≈ 16.33 et on peut estimer à 16.33% le pourcentage
de salariés travaillant à temps partiel par rapport au total des salariés de l'entreprise en 2007.
9°) On veut résoudre l’inéquation y ≥ 20 ⇔ 0.48 x + 8.17 ≥ 20 ⇔ 0.48 x ≥ 11.83 ⇔ x ≥ 24.6 soit x = 25.
Dès 2015, le pourcentage de salariés travaillant à temps partiel par rapport au total des salariés sera d’au
moins 20 %.
10°) Avec la calculette l’équation de la droite de régression par la méthode des moindres carrés de y en
x est donnée par y = 0.5x + 8.01, ce qui est cohérent avec la méthode de Mayer.
IV
A1°. Soit f la fonction définie sur [0,4] par : f ( x ) = 3 + ln(2 x + 2) .
> Appliquons la formule ( ln u ) ' =
u'
avec u = 2x + 2 : on obtient
u
f '( x) = 0 +
x
f ’(x)
> Comme x est dans [0 ;4], x+1 est positif donc
f’(x) est positive sur [0 ;4].
> On en déduit le tableau de variation de f :
2
1
=
.
2x + 2 x +1
0
4
+
3+ln(10) ≈ 5.30
f (x)
3+ln(2) ≈ 3.69
A2a. Soit g la fonction définie sur [1,4] par : g ( x ) = x ² − 4 x + 6 .
> A l’aide des formules classiques, on obtient g '( x ) = 2 x − 4 .
> A l’aide du signe d’une fonction affine, on trouve que
x
0
2
4
g ’= 2x-4
- 0 +
6
6
g (x)
ց
ր
2
Page 4 sur 6
ր
A2b. Voir les tableaux de valeurs (arrondies à 10−2 ) obtenus à l’aide de la calculatrice .
x
f ( x)
g ( x)
0
3.69
6
0,5
4.10
4.25
1
4.39
3
1,5
4.61
2.25
2
4.79
2
2,5
4.95
2.25
3
5.08
3
3,5
5.2
4.25
4
5.30
6
Voir figure en fin de corrigé.
Partie B : Application économique
Une entreprise fabrique un certain type de pièces pour les téléphones mobiles. On admet que pour x milliers de
pièces fabriquées et vendues : la recette, en milliers d'euros, est f ( x ) et le coût total de production, en milliers
d'euros, est g ( x ) .
B1. Graphiquement, l'entreprise réalise un bénéfice quand la courbe recette (Cf) est au dessus de la courbe coût
(Cg) cad pour x compris entre environ 0.5 et 3.8 (milliers de pièces).
Cela correspond à des productions d’environ 500 à 3800 pièces.
B2. Graphiquement, la production x0 pour laquelle le bénéfice est maximal est la valeur de x pour laquelle la
distance entre les courbes recette et coût est maximale (puisque Bénéfice = Recette – Coût).
On lit que cette production est d’environ 2.16 milliers de pièces produites.
B3.
Pour x = 2 : B(2) = 4.79 – 2 = 2.79
Pour x = 2.5 : B(2.5) = 4.95 – 2.25 = 2.7
Le bénéfice maximale est donc d’environ 2.79 milliers d’euros soit 2790 € environ.
y
5
4
benef max
3
2
1
domaine de rentabilité
0
1
2
Page 5 sur 6
3
x
17y
16
15
14
G2
13
12
11
G1
10
9
8
7
6
5
0
1
2
3
4
5
6
7
8
Page 6 sur 6
9
10
11
12
13
14x